Test 1

Pataasin ang iyong marka sa homework at exams ngayon gamit ang Quizwiz!

A patient is prescribed enteric-coated omeprazole [Prilosec] for hyperacidity. What instructions should the nurse provide to the patient regarding medication administration? "Take the medication by chewing or crushing it." "Take this medication 30 to 60 minutes after meals." "Take the medication by dissolving it in water or milk." "Take this medication 30 to 60 minutes before meals."

"Take this medication 30 to 60 minutes before meals." Omeprazole [Prilosec] and other proton pump inhibitors act directly on the proton pump on parietal cells and decrease acid levels. For the drug to be absorbed and show its action, it should be administered at least 30 to 60 minutes before meals. Crushing and chewing the drug will damage its enteric coating and thus should be avoided. The medication can be given by dissolving in water only when the patient has difficulty swallowing and for patients with a nasogastric tube. The medication will have no effect when taken after meals because of the presence of food.

A patient is prescribed enteric-coated omeprazole [Prilosec] for hyperacidity. What instructions should the nurse provide to the patient regarding medication administration? "Take the medication by chewing or crushing it." "Take this medication 30 to 60 minutes after meals." "Take the medication by dissolving it in water or milk." "Take this medication 30 to 60 minutes before meals."

"Take this medication 30 to 60 minutes before meals." Omeprazole [Prilosec] and other proton pump inhibitors act directly on the proton pump on parietal cells and decrease acid levels. For the drug to be absorbed and show its action, it should be administered at least 30 to 60 minutes before meals. Crushing and chewing the drug will damage its enteric coating and thus should be avoided. The medication can be given by dissolving in water only when the patient has difficulty swallowing and for patients with a nasogastric tube. The medication will have no effect when taken after meals because of the presence of food. pp. 954-956

Which instruction should the nurse give a patient who is to take bismuth subsalicylate for eradication of H. pylori? "Nausea and diarrhea are common side effects." "Do not drink alcohol while taking this medication." "The drug can cause your bowel movements to be black." "Take the medication through a straw to prevent staining of your teeth."

"The drug can cause your bowel movements to be black." Bismuth may impart a harmless black coloration to the tongue and the stool, and patients should be forewarned of this effect. Nausea and diarrhea are the most common side effects of clarithromycin [Biaxin], which also is used to treat H. pylori. Alcohol must be avoided when the patient is treated with metronidazole [Flagyl] for H. pylori infection because it may cause a disulfiram-like reaction. Tetracycline, which is also highly active against H. pylori, can cause staining of developing teeth.

A parent brings a child to a primary health care center and reports that the child has abdominal pain, back pain, and fever. The nurse assesses that the child has diminished bowel sounds and a bluish discoloration adjacent to the navel. What does the nurse anticipate the child will be treated for? Pancreatic cyst Acute pancreatitis Pancreatic abscess Chronic pancreatitis

Acute pancreatitis Acute pancreatitis is manifested by fever and abdominal pain radiating to the back. Acute pancreatitis may cause absence of bowel sounds and bluish discoloration in the periumbilical region called Grey Turner's spots. Because the child has bluish discoloration adjacent to the navel, abdominal pain, and fever, the nurse suspects the diagnosis to be acute pancreatitis. Pancreatic cysts and abscesses are not characterized by bluish discoloration adjacent to the navel. Chronic pancreatitis is an inflammation of the pancreas that does not heal or improve with medications and is characterized by bluish discoloration adjacent to the navel.

Which intervention should a nurse implement to reduce discomfort in a patient with acute pancreatitis who complains of discomfort due to abdominal pain? Give 300 mL of water. Administer morphine. Give an abdominal massage. Encourage the patient to go for a walk. Encourage the patient to change position frequently.

Administer morphine. Encourage the patient to change position frequently. Patients with pancreatitis may experience restlessness due to pain in the abdomen. The nurse should administer morphine, which is an analgesic and relieves pain. The nurse should encourage the patient to change positions frequently, which may help relieve the restlessness associated with acute pancreatitis. The nurse should not give water,because patients with acute pancreatitis are on nothing-by-mouth status. Abdominal massage will further aggravate the tenderness. Walking will also aggravate the pain.

A patient who has peptic ulcer disease and is receiving magnesium hydroxide (milk of magnesia) is experiencing an increased number of bowel movements. Which is the nurse's priority action? Encourage the patient to increase dietary fiber. Ask the healthcare provider for a reduction in dose. Instruct the patient to keep an accurate stool count. Administer the drug with an aluminum hydroxide antacid.

Administer the drug with an aluminum hydroxide antacid. Magnesium hydroxide is a rapid-acting antacid with a prominent adverse effect of diarrhea. To compensate, it usually is administered in combination with aluminum hydroxide, which promotes constipation. A reduction in dose might be necessary if the diarrhea is severe, but this is not the priority action. Increasing dietary fiber and keeping a stool count are appropriate actions to implement after adding an antacid to counteract the diarrhea effect.

Which administration technique(s) would be appropriate when giving a sucralfate [Carafate] tablet to a patient with a duodenal ulcer? Break the tablet in half so it is easier to swallow. Administer the tablet with an antacid for maximum benefit. Allow the tablet to dissolve in water before administering it. Administer the tablet with sips of water 1 hour before meals. Crush the tablet into a fine powder before mixing it with water.

Administer the tablet with sips of water 1 hour before meals. Sucralfate acts through a compound that is a sticky gel, which adheres to an ulcer crater, creating a barrier to back-diffusion. The drug is best taken on an empty stomach. The tablet form should not be crushed because crushing it could reduce the effectiveness of the drug. Sucralfate tablets are large and difficult to swallow but can be broken or dissolved in water prior to ingestion. Sucralfate acts under mildly acidic conditions; antacids raise the gastric pH above 4 and may interfere with the effects of sucralfate.

While caring for a patient with pancreatitis, a nurse finds that the patient's blood pressure is 80/70 mm Hg and the nurse suspects hypovolemia. Which treatment option does the nurse expect to be effective? Administration of albumin Administration of octreotide Administration of cyclosporine Administration of acetazolamide

Administration of albumin Acute pancreatitis may cause shock, which is manifested by decreased volume of body fluids and reduced circulatory volume, which results in decreased blood pressure. Albumin is a plasma volume expander that should be administered to help restore the circulatory volume. Octreotide is a drug used to treat esophageal varices in patients with liver cirrhosis. Cyclosporine is a calcineurine inhibitor used as an immunosuppressive drug in patients with liver transplantation. Acetazolamide is a carbonic anhydrase inhibitor used to decrease the volume and bicarbonate concentration of pancreatic secretion in patients with acute pancreatitis.

The nurse is preparing to insert a nasogastric (NG) tube into a 68-year-old female patient who is nauseated and vomiting. The patient has an abdominal mass and suspected small intestinal obstruction. The patient asks the nurse why this procedure is necessary. What response by the nurse is most appropriate? "The tube will help to drain the stomach contents and prevent further vomiting." "The tube will push past the area that is blocked and thus help to stop the vomiting." "The tube is just a standard procedure before many types of surgery to the abdomen." "The tube will let us measure your stomach contents so that we can plan what type of intravenous (IV) fluid replacement would be best."

"The tube will help to drain the stomach contents and prevent further vomiting." The NG tube is used to decompress the stomach by draining stomach contents, and thereby prevent further vomiting. The NG tube will not push past the blocked area. Potential surgery is not indicated currently. The location of the obstruction will determine the type of fluid to use, not measuring the amount of stomach contents. p. 984

The nurse is teaching a group of high school students about the prevention of food poisoning. Which comment by the student shows understanding of foodborne illness protection? "We like to mix up the ingredients so the flavors will melt before we cook our beef stew." "For a snack, I like to eat raw cookie dough from the package instead of baking the cookies." "We only have one cutting board, so we cut up our chicken and salad vegetables at the same time." "When they gave me a pink hamburger I sent it back and asked for a new bun and clean plate."

"When they gave me a pink hamburger I sent it back and asked for a new bun and clean plate." The student who did not accept the pink hamburger and asked for a new bun and clean plate understood that the pink meat may not have reached 160 o F and could be contaminated with bacteria. Mixing ingredients and leaving them long enough for the flavors to melt, eating raw cookie dough from a refrigerated package, and only using one cutting board without washing it with hot soapy water between the chicken and salad vegetables could all lead to food poisoning from contamination. p. 957

A nurse instructs a female patient with peptic ulcer disease who is to start a treatment regimen that includes ranitidine [Zantac] and bismuth subsalicylate [Pepto-Bismol]. Which statement by the patient indicates that the teaching has been effective? "I'll include more calcium and vitamin D in my diet to prevent osteoporosis." "While I'm taking these medications, my bowel movements could look black." "I'm so glad that my allergies will be helped while I'm taking these medications." "I have a medicine at home to take when I start having some loose diarrhea stools."

"While I'm taking these medications, my bowel movements could look black." Regimens for eradicating H. pylori include using two or three antibacterials with an antisecretory agent or histamine 2 receptor antagonist. Bismuth acts topically to disrupt the cell wall of H. pylori. It can cause a harmless black stool discoloration. Loose stools are an adverse effect of systemic antibacterials such as amoxicillin [Amoxil]. Ranitidine [Zantac] produces selective blockade of H 2 receptors, which inhibits gastric acid secretion only, not allergy symptoms. Osteoporosis is an adverse effect of omeprazole, a proton pump inhibitor (PPI). p. 950

The nurse recalls that hepatic coma results primarily from accumulation of which substance in the blood? Sodium Calcium Ammonia Potassium

Ammonia A high ammonia level in the blood is a late manifestation of liver failure that results in hepatic coma, causing neurologic dysfunction and brain damage. Sodium, calcium, and potassium are not directly affected by liver dysfunction or hepatic coma.

biliary cirrhosis

Associated with chronic biliary inflammation and obstruction

A patient has an increased ammonia level associated with hepatic encephalopathy. What assessment finding does the nurse expect? Aphasia Asterixis Hyperactivity Acute dementia

Asterixis Asterixis is a twitching spasm of the hands and wrists seen in patients with increased ammonia levels in conditions such as hepatic encephalopathy. Aphasia, hyperactivity, and acute dementia are manifestations not associated with hepatic encephalopathy. Besides asterixis, an increased serum ammonia level causes sedation and confusion that progress to a comatose state. p. 1021

A nurse is teaching a patient about prevention of peptic ulcers. What instructions should the nurse give the patient? Avoid smoking. Consume raw uncooked food. Use nonsteroidal antiinflammatory drugs (NSAIDs) for treatment of pain. Wash hands thoroughly with soap after using the restroom and before eating. Report symptoms of gastric irritation, such as nausea and epigastric pain, to the health care provider.

Avoid smoking. Wash hands thoroughly with soap after using the restroom and before eating. Report symptoms of gastric irritation, such as nausea and epigastric pain, to the health care provider. p. 946

What is the name of the substance that is a mixture of cholesterol crystals and calcium salts that may be found in a patient diagnosed with pancreatitis? Apraxia Biliary sludge Biliary atresia Pancreatic abscess

Biliary sludge Biliary sludge is a mixture of cholesterol crystals and calcium salts and is seen in patients with bile stasis. Apraxia is the inability to construct simple figures. Biliary atresia refers to congenital biliary abnormalities. Pancreatic abscess is a condition associated with the collection of pus in the pancreas.

What surgical procedure involves the removal of two-thirds of the stomach and anastomosis of the gastric stump to the duodenum? Vagotomy Pyloroplasty Billroth I operation Billroth II operation

Billroth I operation A Billroth I operation, also known as a gastroduodenostomy, is a partial gastrectomy in which the distal two thirds of the stomach is removed and there is an anastomosis of the gastric stump to the duodenum. A vagotomy is the severing of the vagus nerve totally or selectively. A pyloroplasty is the surgical enlargement of the pyloric sphincter in order to facilitate easy passage of contents from the stomach. A Billroth II operation is called a gastrojejunostomy; it is the removal of the distal two thirds of the stomach and anastomosis of the gastric stump to the jejunum. p. 949

A patient states, "I always get diarrhea when I am traveling." Which medication does the nurse expect will be beneficial to the patient? Dextranomer Metronidazole Methylcellulose Bismuth subsalicylate

Bismuth subsalicylate Traveler's diarrhea is common when traveling to areas with poor sanitation. Bismuth subsalicylate is an anti-diarrheal drug that decreases the secretions, has weak antibacterial activity, and is beneficial for traveler's diarrhea. Dextranomer is used in the treatment of fecal incontinence. Metronidazole is beneficial for a patient who has diarrhea because of antibiotics. Methylcellulose is a bulk-forming laxative and is beneficial for a patient with constipation. This medication does not help relieve traveler's diarrhea. p. 963

diet pancreatitis

Bland, low-fat Small, frequent meals No alcohol or caffeine

The nurse finds coffee ground-like granules in the nasogastric drainage of a patient. What does the nurse suspect to be the cause of this condition? Presence of bile Possibility of hemorrhage Blood modified by acidic gastric secretions Displacement of the tube to the upper gastrointestinal tract

Blood modified by acidic gastric secretions The presence of coffee ground-like granules in the drainage indicates that acidic gastric secretions have modified the patient's blood. Gastric secretions oxidize the iron in the blood and result in coffee ground-like granules. A greenish tinge in the drainage indicates the presence of bile. A bright red color of drainage indicates hemorrhage. The drainage from the tube will be dark red if the tube has entered the gastrointestinal tract. p. 971

A patient with gastroesophageal reflux disease (GERD) is on cimetidine therapy. Which parameter does the nurse monitor to provide effective care? Bowel sounds Motor movements Serum calcium levels Serum magnesium levels

Bowel Sounds Cimetidine is a histamine-receptor used in the treatment of peptic ulcer and GERD. Cimetidine decreases gastric motility and causes constipation. Therefore, the nurse monitors the patient for any changes in bowel sounds. This may help with early detection of constipation. Cimetidine does not have extrapyramidal side effects and does not cause motor incoordination. Cimetidine does not alter serum calcium or magnesium levels. p.934

How do H 2 receptor antagonists reduce the secretion of acids? By blocking the H 2 receptors By stimulating gastrin receptors By blocking acetylcholine receptors By stimulating acetylcholine receptors

By blocking the H 2 receptors H2 receptor antagonists block H 2 receptors and stop the secretion of acids by parietal cells. H 2 receptor antagonists do not block acetylcholine receptors. They make parietal cells unresponsive to the stimulation of acetylcholine and gastrin receptors. H 2 receptor antagonists do not directly affect gastrin and acetylcholine receptors. pp. 951-952

duodenal ulcer risk factors

COPD Cirrhosis of liver Chronic pancreatitis Hyperparathyroidism Chronic kidney disease Zollinger-Ellison syndrome

What does a nurse anticipate that the primary health care provider will prescribe for a patient with pancreatitis who reports irritability and tingling around the lips? Insulin Vitamin K Cholestyramine Calcium gluconate

Calcium gluconate Jerking movements, tingling around the lips, and irritability are symptoms of tetany, which is caused by hypocalcemia. When calcium levels drop, the salt form of calcium is administered; therefore, the nurse expects the primary health care provider to prescribe calcium gluconate. Insulin is an antidiabetic drug and is prescribed in case of hyperglycemia. Vitamin K is an anticoagulant and is administered to patients with coagulation abnormalities. Cholestyramine belongs to the bile acid sequestrant and is prescribed to remove bile from the body through the feces.

What amount is considered "A massive upper GI hemorrhage" (mL & %)

1500 or more mL of blood or 25% of intravascular blood volume

Postexposure prophylaxis: Hepatitis A, B, & C

A: vaccine and immune globulin (IG) B: vaccine and hepatitis B immune globulin (HBIG) C: no post exposure prophylaxis, baseline & follow up testing to monitor

pain associated w/ acute pancreatitis, chronic pancreatitis

Acute & Chronic: Left upper quadrant or midepigastrium Radiates to the back Sudden onset Deep, piercing, continuous or steady Aggravated by eating Starts when recumbent Not relieved with vomiting Chronic only: Heavy, gnawing feeling; burning and cramplike

at least how many mL/kg/hr indicates adequate renal perfusion?

0.5 mL/kg/hr

What advice should a nurse give to a patient who was prescribed omeprazole [Prilosec] for peptic ulcers? "Eat foods that are rich in magnesium." "Take omeprazole [Prilosec] only at bedtime." "Take aspirin along with omeprazole [Prilosec]." "Take dietary supplements containing vitamin D."

"Eat foods that are rich in magnesium." Omeprazole [Prilosec] is a proton pump inhibitor (PPI). PPIs come with the warning that they deplete magnesium. Therefore, the nurse should advise the patient to eat magnesium-rich foods to combat the adverse effect associated with omeprazole [Prilosec]. Taking vitamin D supplements along with omeprazole [Prilosec] does not have any therapeutic benefit unless taken along with calcium supplements. A patient with peptic ulcers should not take aspirin along with omeprazole [Prilosec]; this combination will aggravate the situation because aspirin is a nonsteroidal antiinflammatory drug that can cause peptic ulcers. Omeprazole [Prilosec] works better when taken before meals, not before bedtime.

Which statement indicates a need for further teaching by a student nurse about nutritional therapy in patients with acute pancreatitis? "I should feed the patient through a nasojejunal tube." "I should monitor the patient's blood triglyceride levels." "I should give small quantities of food frequently to the patient." "I should include fat-rich food substances in the patient's dietary plan."

"I should include fat-rich food substances in the patient's dietary plan." Patients with acute pancreatitis should be provided with food that does not stimulate the pancreas much. Fats are broken down by the enzymes produced by pancreas; therefore, fat-rich food substances should not be given to the patient. Patients with pancreatitis are given nasojejunal tube feeding. Patients with pancreatitis have malfunctioning of the pancreas and therefore do not metabolize fats. If fat-rich substances or intravenous lipids are given, then the blood triglyceride level may rise. Therefore the nurse should monitor the patient's blood triglyceride levels. To avoid burden on the pancreas, feedings should be given frequently in small quantities.

The nurse is teaching an older patient about treatment and management of peptic ulcer disease. Which statement made by the patient indicates the need for further teaching? "I should report abdominal pain or discomfort to the nurse." "I should avoid irritating substances that cause peptic ulcer." "I should take nonsteroidal antiinflammatory drugs (NSAIDs) with food, milk, or antacids." "I should adhere to proton pump inhibitor (PPI) and histamine-receptor blocker therapy as prescribed."

"I should report abdominal pain or discomfort to the nurse." The patient should report abdominal pain or discomfort to the primary health care provider, not to the nurse. For the treatment and management of peptic ulcer disease the nurse should instruct the patient to avoid irritating substances that cause peptic ulcer; take nonsteroidal antiinflammatory drugs (NSAIDs) and other gastric irritating substances along with food, milk, or antacids; and adhere to the proton pump inhibitor (PPI) and histamine-receptor blocker therapy as prescribed.

The nurse is evaluating a patient after teaching about management of peptic ulcer disease. Which statement by the patient indicates the need for further teaching? "I should report increased vomiting or epigastric pain." "I should avoid smoking, because it may delay healing of the ulcer." "I should avoid spicy and acidic food that may cause epigastric distress." "I should take medications that include only antisecretory class of drugs." "I should take over-the-counter drugs that have ingredients like aspirin."

"I should take medications that include only antisecretory class of drugs." "I should take over-the-counter drugs that have ingredients like aspirin." p. 949

Urine specific gravity normal value

1.005 to 1.025-Consistent readings greater than 1.025 indicate that the urine is extremely concentrated and that blood volume is probably low

A patient is scheduled for a secretin stimulation test and asks the nurse what this will evaluate. What is the best response by the nurse? "It will evaluate if you have Wilson's disease." "It will evaluate if you have diabetes mellitus." "It will determine if you have acute pancreatitis." "It will determine if you have chronic pancreatitis."

"It will determine if you have chronic pancreatitis." The secretin stimulation test is performed in patients with chronic pancreatitis to determine the degree of pancreatic function. A test to check serum ceruloplasmin level is ordered in patients with Wilson's disease. A diagnosis of diabetes mellitus is determined by blood glucose levels in the body, not by the secretin stimulation test. Acute pancreatitis is diagnosed by a contrast-enhanced CT scan.

A nurse should give which nonpharmacologic instruction to a patient who has peptic ulcers? "Reduce your intake of caffeinated beverages such as coffee and colas." "Take a nonsteroidal anti-inflammatory drug once a day to help with pain." "An ulcer diet of bland foods with milk and cream products will speed healing." "It would be better to eat five or six small meals a day instead of three larger ones."

"It would be better to eat five or six small meals a day instead of three larger ones." One optimal, nondrug measure, in addition to drug management, to aid patients with peptic ulcers is changing the eating pattern to more frequent, smaller meals to avoid fluctuations in intragastric pH. No evidence indicates that caffeinated beverages promote ulcer formation or that an "ulcer diet" improves healing. Nonsteroidal anti-inflammatory drugs inhibit the biosynthesis of prostaglandins, which reduce mucosal blood flow and promote the secretion of gastric acid.

The nurse provides a patient with educational materials about antacids. Which statements about antacids are appropriate? Calcium-containing antacids cause constipation. Antacids form a protective barrier in the stomach. Aluminum-containing antacids cause constipation. Antacids neutralize the acid present in the stomach. Magnesium-containing antacids cause constipation. Antacids decrease the secretion of acid in the stomach.

Calcium-containing antacids cause constipation. Aluminum-containing antacids cause constipation. Antacids neutralize the acid present in the stomach. Both calcium- and aluminum-containing antacids cause constipation as an adverse effect. Antacids neutralize the excess acid secreted in the stomach by forming salts. Agents protective against ulcers, such as sucralfate [Carafate], form a mucous barrier in the stomach; the antacids do not. Antacids can only neutralize the acid secreted in the stomach; they cannot influence the secretion of acids. Magnesium-containing antacids reduce the effect of constipation resulting from aluminum- and calcium-containing antacids.

The patient with suspected pancreatic cancer is having many diagnostic studies done. Which one can be used to establish the diagnosis of pancreatic adenocarcinoma and for monitoring the response to treatment? Abdominal ultrasound Spiral computed tomography (CT) scan Cancer-associated antigen 19-9 (CA 19-9) A positron emission tomography (PET)/CT scan

Cancer-associated antigen 19-9 (CA 19-9) CA 19-9 is the tumor marker used for the diagnosis of pancreatic adenocarcinoma and for monitoring the response to treatment. Although a spiral CT scan may be the initial study done and provides information on metastasis and vascular involvement, this test and the PET/CT scan or abdominal ultrasound do not provide additional information.

What happens in cirrhosis?

Cells attempt to regenerate but are disorganized Results in abnormal architecture, overgrowth of fibrous connective tissue, and impeded blood flow

F & E imbalances associated w/ acute pancreatitis

Chloride, sodium, and potassium Hypocalcemia :Tetany, Calcium gluconate to treat Hypomagnesemia

A patient reports severe abdominal pain. A primary health care provider diagnoses it to be chronic pancreatitis secondary to cholecystitis. Which treatment option does the nurse anticipate will be beneficial? Cholecystectomy Whipple procedure Chemoembolization Choledochojejunostomy

Choledochojejunostomy When a patient with chronic pancreatitis is diagnosed with a biliary tract disease, surgery is required to treat the condition. The surgery involves diversion of bile flow and relieving the duct from obstruction. In patients with chronic pancreatitis, choledochojejunostomy is performed to divert bile flow around the ampulla of Vater. Therefore the nurse anticipates that choledochojejunostomy will be beneficial to the patient. Cholecystectomy is performed in patients with gallbladder disease. A Whipple procedure is the treatment option for patients with pancreatic cancer. Chemoembolization is performed in patients with liver cancer.

Measures for relief or pruritis (common manifestation of cirrhosis)

Cholestyramine or hydroxyzine Baking soda or Alpha Keri baths Lotions, soft or old linen Temperature control Short nails; rub with knuckles

A patient complains of burning pain in the abdomen four hours after eating a meal. Upon assessment, the nurse identifies the pain in midepigastric region. Which risk factors are associated with the pain? Bile reflux Chronic pancreatitis Chronic kidney disease Zollinger-Ellison syndrome Incompetent pyloric sphincter

Chronic pancreatitis Incompetent pyloric sphincter p. 944

A patient with hepatitis A is in the acute phase. The nurse plans care while anticipating that the patient may be experiencing which symptoms? Fatigue Pruritus Anorexia Dizziness Visual disturbances

Fatigue Pruritus Anorexia The acute phase of hepatitis usually lasts from one to four months. During the incubation period, symptoms may include malaise, anorexia and weight loss, fatigue, nausea, occasional vomiting, and abdominal (right upper quadrant) discomfort. The patient may find food repugnant, and smokers may have distaste for cigarettes. There is also a decreased sense of smell. Other symptoms may include headache, low-grade fever, arthralgias, and skin rashes. Pruritus (intense chronic itching) sometimes accompanies jaundice. The pruritus occurs as a result of the accumulation of bile salts beneath the skin. Dizziness and visual disturbances are not symptoms of the acute phase of hepatitis A.

Position of comfort in acute pancreatitis

Flex trunk and draw knees to abdomen Side-lying with head of bed elevated 45 degrees

diagnostic studies CRC

Flexible sigmoidoscopy every 5 years Colonoscopy every 10 years Double-contrast barium enema study every 5 years CT colonography every 5 years Annually: Fecal occult blood test (FOBT) , Fecal immunochemical test (FIT), Test for blood in the stool, Must be done frequently to catch intermittent bleeding common with tumors colonoscopy is gold standard: entire colon is examed, Biopsy samples can be obtained, Polyps can be immediately removed and sent to the laboratory for examination CBC to check for anemia Coagulation studies Liver function tests CT scan or MRI of the abdomen are helpful in detecting-Liver metastases, Retroperitoneal and pelvic disease, & Depth of penetration of tumor in bowel wall

clinical manifestations- acute pancreatitis

Flushing Cyanosis Dyspnea Nausea/vomiting Low-grade fever Leukocytosis Hypotension, tachycardia Jaundice Abdominal tenderness with guarding Decreased or absent bowel sounds Crackles Abdominal skin discoloration Grey Turner's spots or sign Cullen's sign Shock

A patient with type 2 diabetes mellitus has been diagnosed with non-alcoholic fatty liver disease (NAFLD). The nursing teaching plan should include which of the following? Having genetic testing done Eliminating carbohydrates from the diet Following measures to gain tighter glucose control Avoiding alcohol until liver enzymes return to normal

Following measures to gain tighter glucose control NAFLD can progress to liver cirrhosis. There is no definitive treatment, and therapy is directed at reduction of risk factors, which include treatment of diabetes, reduction in body weight, and elimination of harmful medications. For those who are overweight, weight reduction is important. Weight loss improves insulin sensitivity and reduces liver enzyme levels. NAFLD does not show up positive on a genetic test. It is not recommended to completely eliminate carbohydrates from the diet. NAFLD is not caused by alcohol, and the question does not imply that the patient drinks.

how does pH increases to 3.5 or more?

Food or antacids neutralize stomach acid or Drugs block acid secretion

The nurse cares for a patient with advanced cirrhosis. What indicates that the patient is experiencing a serious complication? Urine retention Increased blood glucose No bowel movement in three days Frequent nosebleeds and bruising

Frequent nosebleeds and bruising The liver produces clotting factors. As cirrhosis becomes more advanced, the production of clotting factors is disrupted and thereby decreased, making the patient more susceptible to bleeding. Increasing frequency and severity of nosebleeds and bruising would indicate a deterioration in liver function. Urine retention, abnormal blood glucose, and constipation are not directly associated with advanced cirrhosis.

cardiac cirrhosis

From long-standing severe right-sided heart failure

What are the characteristics of gastric ulcers? Gastric ulcer lesions are superficial. Gastric ulcers increase the gastric secretion. Gastric ulcers predominantly occur in antrum. Gastric ulcers cause pain one to two hours after meals. Gastric ulcers manifest as burning, cramping pain in the midepigastrium.

Gastric ulcer lesions are superficial. Gastric ulcers predominantly occur in antrum. Gastric ulcers cause pain one to two hours after meals. Gastric ulcer lesions are superficial, round, oval, or cone shaped. They predominantly occur in antrum and cause pain one to two hours after meals. Gastric ulcers decrease gastric secretion, whereas duodenal ulcers increase gastric secretion. In gastric ulcers, there is a burning, cramping pain in the high left epigastrium. p. 943

stomach cancer risk factors

Gender -- men have more than double the risk than women Race -- African-American or Asian may increase risk Genetics -- genetic abnormalities and some inherited cancer syndromes may increase risk Blood type -- individuals with blood group A may be at increased risk Advanced age -- stomach cancer occurs more often around ages 70 and 74 in men and women, respectively. Family history of gastric cancer can double or triple risk of stomach cancer Smoking, drinking alcohol, and eating a diet low in fruits and vegetables or high in salted, smoked, or nitrate-preserved foods H. pylori infection of the stomach Chronic gastritis, pernicious anemia, gastric polyps Coal mining, nickel refining, rubber and timber processing, and asbestos exposure

Which infection does the nurse suspect in a patient with periodontitis who complains of bleeding during brushing and reveals the he or she has a high intake of soft foods? Parotitis Gingivitis Stomatitis Oral candidiasis

Gingivitis p. 928

During follow-up visits, a nurse finds that the patient is having an exacerbation of pancreatitis. Which action of the patient's caregiver is responsible for this condition? Giving Creon along with meals Giving three large meals a day Checking for fatty stools in the patient Giving omeprazole one hour after meals

Giving three large meals a day Patients with pancreatitis should be given small and frequent meals to reduce or prevent pancreatic stimulation. Therefore avoiding small frequent meals is responsible for the patient's condition. Creon is a pancreatic enzyme supplement to be given along with meals or snacks. Pancreatitis is associated with fatty and foul-smelling stools; monitoring of stools is required in patients with this disease. Omeprazole is an antisecretory agent used for reducing gastric acid in the stomach; it is given after meals to increase its duration of action.

A nurse monitors a patient who has peptic ulcer disease and is taking antibacterial medications. If the treatment has been effective, the patient's breath test result should reveal the absence of what? H. pylori Histamine 2 Bicarbonate Prostaglandins

H. pylori Antibacterial medications should be given to patients who have confirmed infection with H. pylori, a cause of ulcers. A breath test is a noninvasive means of measuring H. pylori levels. The patient is given radiolabeled urea, which converts to carbon dioxide and ammonia if H. pylori is present. Radiolabeled carbon dioxide then can be detected in the breath. Bicarbonate, histamine 2, and prostaglandins are not affected by the actions of antibacterial medications for peptic ulcers.

drug therapy PUD

H2 receptor blockers, PPIs, antibiotics, antacids (adjunct therapy)

Which condition does the nurse expect in a patient with gastroesophageal reflux disease (GERD) who has esophagitis and stenosis? Achalasia Hiatal hernia Esophageal strictures Eosinophilic esophagitis

Hiatal hernia GERD associated with esophagitis and stenosis leads to a hiatal hernia. A hiatal hernia is a herniation of a portion of the stomach into the esophagus through a hiatus in the diaphragm. Achalasia is the absence of peristalsis in the lower two-thirds of the esophagus, characterized by nocturnal regurgitation, dysphagia, and foul-smelling breath. Esophageal stricture is the narrowing of the esophagus, which involves dysphagia, regurgitation, and weight loss. Eosinophilic esophagitis is the inflammation of the esophagus due to an infiltration of eosinophils. p. 936

Which increases the concentration of gastric acids? Hypersecretion of parietal cells Inhibition of proton pump activity Inhibition of acetylcholine receptors Hypersecretion of proteolytic enzymes

Hypersecretion of parietal cells Hydrochloric acid, which is necessary for digestion, is secreted by the parietal cells lining the stomach. Hypersecretion of these cells causes acidity or more severe disorders. Inhibition of the proton pump reduces the hypersecretion of acid. Proteolytic enzymes such as pepsinogen, which is activated by hydrochloric acid, are secreted by the chief cells of the stomach and are responsible for the cleaving of proteins. Acetylcholine receptors also mediate the proton pump, and inhibition of these receptors helps to prevent the effects caused by a highly acidic environment

Diet hepatitis

Increased calorie Low fat if not tolerated Vitamin supplements avoid alcohol intake/ other drugs detoxified by liver

For someone w/ chronic pancreatitis, when is surgery indicated? What procedure will be done?

Indicated when biliary disease is present or if obstruction or pseudocyst develops Diverts bile flow or relieves ductal obstruction Choledochojejunostomy Roux-en-Y pancreatojejunostomy

Diverticulitis

Inflammation and/or infection associated with diverticula Peritonitis caused by rupture of a pericolic or pelvic abscess Hemorrhage d/t rupture; Diverticular bleeding is arterial Treated with bowel rest and intravenous antibiotics When a diverticular phlegmon or abscess extends or ruptures into an adjacent organ, fistulas may occur.

A patient is admitted to the hospital with a severe duodenal ulcer. The patient suddenly complains of severe pain spreading over the entire abdomen, likely due to a perforation. What should be the most immediate intervention by the nurse, if prescribed? Administer nitrates. Administer pain medication. Prepare for laparoscopic surgery. Insert a nasogastric (NG) tube into the stomach.

Insert a nasogastric (NG) tube into the stomach. The immediate focus of management for a patient with a perforation is to stop the spillage of gastric or duodenal contents into the peritoneal cavity and restore blood volume. An NG tube is inserted into the stomach to provide continuous aspiration and gastric decompression to stop spillage through the perforation and thereby prevent peritonitis. Administering nitrates to such a patient will not be helpful in relieving the condition. Administration of pain medications and preparations for laparoscopic surgery are done later.

The patient with advanced cirrhosis asks why his or her skin is so yellow. The nurse's response is based on what knowledge? Decreased peristalsis in the gastrointestinal tract contributes to a buildup of bile salts. Jaundice results from the body's inability to conjugate and excrete bilirubin. A lack of clotting factors promotes the collection of blood under the skin surface. Decreased colloidal oncotic pressure from hypoalbuminemia causes the yellowish skin discoloration.

Jaundice results from the body's inability to conjugate and excrete bilirubin. p. 986

A patient reports vomiting blood. Upon further analysis, the patient is diagnosed with increased production of gastric acid. Which drug should the nurse anticipate being included in the treatment plan? Famotidine [Pepcid] Misoprostol [Cytotec] Simethicone [Mylicon] Lansoprazole [Prevacid]

Lansoprazole [Prevacid] Lansoprazole receptors to histamine, but it has no effect on gastrin receptors. Simethicone [Mylicon] is an antiflatulent drug and has no effect on the secretion of acid in the stomach.

Which condition does the nurse suspect in a patient with oral cancer who has "smoker's patch" on the mucosa of the mouth? Pyrosis Leukoplakia Erythroplasia Hyperkeratosis

Leukoplakia p. 929

S/S of shock

Low BP Rapid, weak pulse Increased thirst Cold, clammy skin Restlessness Monitor the patient's vital signs every 15 to 30 minutes, and inform the health care provider of any significant changes. Use caution when administering sedatives for restlessness: Warning sign of shock may be masked by drugs

A patient reports nausea, vomiting, and abdominal tenderness. The patient's laboratory reports reveal an erythrocyte sedimentation rate of 32 mm/hr, a white blood cell (WBC) count of 12,000/-µL, 2.2 mg/dL of bilirubin, and serum alkaline phosphatase of 120 U/L. What does the nurse suspect that this indicates? Cirrhosis Gallstones Liver cancer Chronic pancreatitis

Chronic pancreatitis Pancreatitis is manifested by abdominal pain, nausea, vomiting, jaundice, leukocytosis, increased erythrocyte sedimentation rate, and elevated levels of serum alkaline phosphatase. The normal WBC count, bilirubin, and alkaline phosphatase are 4,500 to 10,000 cells/-µL, 0.3 to 1.9 mg/dL, and 25 to 100 U/L, respectively. The patient's findings reveal leukocytosis, an increased erythrocyte sedimentation rate, and an elevated serum alkaline phosphatase. Therefore the nurse suspects that the patient has chronic pancreatitis. Cirrhosis is associated with decreased serum albumin and increased serum globulin, bilirubin, and prolonged prothrombin time. Liver enzymes and serum amylase are elevated in the patient with gallstones. Liver cancer is associated with increased serum bilirubin and alpha-fetoprotein.

An elderly, critically ill patient who was hospitalized underwent gastric surgery 1 week ago. The caregiver reports that the patient has seemed confused for the past 2 to 3 days. Which drug would most likely have caused this symptom in the patient? Sucralfate [Carafate] Misoprostol [Cytotec] Cimetidine [Tagamet] Pantoprazole [Protonix]

Cimetidine [Tagamet] Cimetidine [Tagamet] is an H 2 receptor antagonist that is associated with confusion and disorientation as adverse effects in elderly patients. Sucralfate [Carafate] is an agent that protects against ulcers. It is not prescribed for critically ill patients. Misoprostol [Cytotec] is a prostaglandin analogue associated with adverse effects such as gastrointestinal distress, bleeding in the vagina, and headache. Pantoprazole [Protonix] is a proton pump inhibitor associated with gastric tumors.

A nurse assesses a male patient who has developed gynecomastia while receiving treatment for peptic ulcers. Which medication from the patient's history should the nurse recognize as a contributing factor? Amoxicillin [Amoxil] Cimetidine [Tagamet] Omeprazole [Prilosec] Metronidazole [Flagyl]

Cimetidine [Tagamet] Cimetidine binds to androgen receptors, producing receptor blockade, which can cause enlarged breast tissue (gynecomastia), reduced libido, and impotence. All these effects reverse when dosing stops. Amoxicillin, metronidazole, and omeprazole are not associated with gynecomastia.

A patient with cancer of the head of the pancreas is admitted to the hospital. What are the manifestations that a nurse might expect to find in this patient? Clay-colored stools Itching and irritation of the skin Ulcers on the back and abdomen Swelling of the face and extremities Extreme pain in the upper abdomen that may radiate to the back

Clay-colored stools Itching and irritation of the skin Extreme pain in the upper abdomen that may radiate to the back Tumor of the head of the pancreas will obstruct the common bile duct where it passes through the head of the pancreas to join the pancreatic duct and empty at the ampulla of Vater into the duodenum. The stools will be clay-colored when bile is not able to enter the duodenum. Pruritus is also a common symptom in cancer of the pancreas; hence the patient may complain of itching and irritation of the skin. Severe pain is also present. The pain generally depends on the part affected and severity. Edema and ulcers are not common manifestations of pancreatic cancer.

The nurse is reviewing the prescriber's orders and notes that omeprazole [Prilosec] has been ordered for a patient admitted with acute coronary syndrome (ACS). The nurse should be concerned if this medication is combined with which medication noted on the patient's record? Aspirin 81 mg daily Clopidogrel [Plavix] 75 mg daily Metoprolol 50 mg every 8 hours Heparin 5000 units subcutaneous every 12 hours

Clopidogrel [Plavix] 75 mg daily For patients who lack risk factors for gastrointestinal (GI) bleeding, combined use of clopidogrel with a proton pump inhibitor (PPI) such as omeprazole may reduce the effects of clopidogrel without offering any real benefits and thus should be avoided. This is due to inhibition of CYP2C19, which converts the drug to its active form. Nothing in the question indicates that the patient is at risk for GI bleeding. Coadministration with aspirin, metoprolol, or heparin is not cause for concern. p. 955

what to monitor from when Hemorrhage is result of chronic alcohol abuse

Closely observe for delirium tremens -Agitation -Uncontrolled shaking -Sweating -Vivid hallucinations

Diagnostic studies: upper GI bleed

Complete blood cell count (CBC) Blood urea nitrogen (BUN) measurement Serum electrolyte measurements Prothrombin time, partial thromboplastin time Liver enzyme measurements ABG measurements Typing/crossmatching for possible blood transfusions Arterial blood gases (ABGs) may be measured, and typing and crossmatching for possible blood transfusions may also be done. All vomitus and stools should be tested for gross and occult blood. The initial hematocrit may be normal and may not reflect the loss until 4 to 6 hours after fluid replacement, because initially the losses of plasma and RBCs are equal. During a significant hemorrhage, blood proteins are broken down by GI tract bacteria, resulting in elevated BUN levels. Urinalysis- Specific gravity: indication of patient's hydration status

A patient underwent pancreaticoduodenectomy (Whipple procedure) for pancreatic cancer. What are the instructions that the nurse should include when giving dietary advice to this patient? Consume a low-fat diet. Consume a high-calorie diet. Consume a diet high in proteins. Consume a low-carbohydrate diet. Patient may resume normal diet without any restrictions.

Consume a low-fat diet. Consume a high-calorie diet. Consume a diet high in proteins. Having a low-fat, high-carbohydrate, and high-protein diet is essential. The diet should be low in fat to decrease the work load of the liver and promote healing. A high-calorie diet should be provided because more energy is required. High-calorie meals are needed for energy and to promote the use of protein for tissue repair. A high-protein diet is required for tissue building. The diet should be high in carbohydrates to provide the required energy. The patient should not resume a normal diet and should follow the restrictions as advised.

Hep B transmission & at risk populations

DNA virus transmitted: Perinatally, Percutaneously , Via mucosal exposure to infectious blood, blood products, or other body fluids Those practicing unprotected anal intercourse Household contact of chronically infected Patients undergoing hemodialysis Health care and public safety workers Transplant recipients

While caring for a patient with pancreatitis, a nurse finds that the patient's blood pressure dropped to 90/60 mm Hg from an initial baseline of 125/80 mm Hg. The nurse administers isotonic fluid. Which medication should the nurse administer as prescribed by the primary health care provider? Morphine Dopamine Omeprazole Pancrelipase

Dopamine Vasoactive drug used for Ongoing hypotension pp. 1000-1001

A patient arrives in the emergency department with severe acute pancreatitis and a blood pressure of 80/40 mm Hg and heart rate of 142. What medication does the nurse anticipate administering to this patient? Morphine Dopamine Dicyclomine Nitroglycerine

Dopamine Shock is associated with decreased blood pressure. Dopamine is a vasoconstrictor drug that increases peripheral vascular resistance, thereby increasing blood pressure; therefore, dopamine is used to treat shock. Morphine is an analgesic used to relieve pain. Dicyclomine is an antispasmodic used to treat muscle spasms. Nitroglycerine is a vasodilator used as a smooth muscle relaxant. p. 1032

Common causes of upper GI bleeds

Drug Induced • Corticosteroids • Nonsteroidal antiinflammatory drugs (NSAIDs) • Salicylates Esophagus • Esophageal varices • Esophagitis • Mallory-Weiss tear Stomach and Duodenum • Stomach cancer • Hemorrhagic gastritis • Peptic ulcer disease • Polyps • Stress-related mucosal disease Systemic Diseases • Blood dyscrasias (e.g., leukemia, aplastic anemia) • Renal failure

The patient is having a gastroduodenostomy (Billroth I operation) for stomach cancer. What long-term complication is occurring when the patient reports generalized weakness, sweating, palpitations, and dizziness 15 to 30 minutes after eating? Malnutrition Bile reflux gastritis Dumping syndrome Postprandial hypoglycemia

Dumping syndrome After a Billroth I operation, dumping syndrome may occur 15 to 30 minutes after eating because of the hypertonic fluid going to the intestine and additional fluid being drawn into the bowel. Malnutrition may occur, but does not cause these symptoms. Bile reflux gastritis cannot happen when the stomach has been removed. Postprandial hypoglycemia occurs with similar symptoms, but two hours after eating. p. 953

The nurse is obtaining a health history from a patient who comes to the office for evaluation of gastric distress. The patient indicates that the symptoms occur two to five hours after meals, and the pain is "burning" and sometimes like a cramp in the midepigastric region, just below the xiphoid process. Based on these descriptions, the nurse suspects that the patient has which disorder? Esophagitis Gastric ulcer Duodenal ulcer Chronic gastritis

Duodenal ulcer p. 943

The student nurse is providing postoperative care to a patient who underwent ostomy surgery. Which action of the student nurse indicates the need for correction? Using open-ended and transparent pouch Emptying collecting bags when filled completely Placing a pouch on irritated skin using a skin barrier Expelling flatus from the bag with a charcoal filter

Emptying collecting bags when filled completely The weight of drainage from the stoma pulls the skin barrier away from the skin and loosens the seal. Therefore, the nurse should empty the ostomy bag when it is one-third full, instead of when it is completely full. Using open-ended and transparent pouches helps the nurse to observe the stoma and collect the drainage. Placing a pouch on irritated skin using a skin barrier is done to prevent harm to the area. Expelling flatus from the bag with a charcoal filter helps to control odor. p. 992

most accurate diagnostic procedure for ulcers

Endoscopy allows for direct viewing of the gastric and duodenal mucosa (Fig. 42-14). It can be used to determine the degree of ulcer healing after treatment and to confirm the absence of malignancy. During endoscopy, tissue specimens are obtained for identification of H. pylori and to rule out stomach cancer.

A nurse assesses a patient with suspected peptic ulcer disease. Which symptom will the patient most likely report? Vomiting after meals Abdominal distention after eating Intolerance of fatty and spicy foods Epigastric discomfort relieved by eating

Epigastric discomfort relieved by eating Symptoms of peptic ulcer disease (PUD) are variable and often absent. However, discomfort, if present, may occur before meals or two to three hours after meals and at bedtime. The discomfort may be relieved by eating because the food will dilute and buffer gastric acid. Although vomiting or abdominal distention after meals may occur, they are less likely to be associated with PUD than is the relief caused by eating.

Drug therapy for bleeding r/t ulceration

Epinephrine Produces tissue edema → pressure on bleeding source Usually combined with other therapies

While caring for a patient with chronic pancreatitis, a primary health care provider orders the nurse to administer nortriptyline. What beneficial effect of nortriptyline will the nurse monitor in this patient? Reduction in neuropathic pain Normalization of blood glucose Decreased nausea and vomiting Improved digestion of fatty foods

Reduction in neuropathic pain Nortriptyline is an antidepressant that is effective in reducing neuropathic pain in patients with chronic pancreatitis. Chronic pancreatitis is also associated with hyperglycemia, and insulin is injected to correct the increased blood glucose levels. Gastric acid secretions, nausea, and vomiting are reduced by antacids such as omeprazole. Patients with chronic pancreatitis have a deficiency of pancreatic enzymes. Products such as pancrelipase and Creon are prescribed to improve the digestion of fatty foods.

The nurse is caring for a patient diagnosed with gastric outlet obstruction. Which assessment findings support the diagnosis? Rigid board-like abdomen Audible abdominal sounds Continuous epigastric distress Overtime dilation of the stomach Visible swelling in the upper abdomen

Rigid board-like abdomen Visible swelling in the upper abdomen Overtime dilation of the stomach and visible swelling in the upper abdomen occur as a result of gastric outlet obstruction due to edema, inflammation, and fibrous scar tissue formation. A rigid board-like abdomen is indicative of perforated ulcer. Audible abdominal sounds are clinical manifestations of dumping syndrome. Continuous epigastric distress occurs after meals in patients with bile reflex gastritis.

A patient with a history of peptic ulcer disease has presented to the emergency department reporting severe abdominal pain and has a rigid, board-like abdomen that prompts the health care team to suspect a perforated ulcer. What intervention should the nurse anticipate? Providing intravenous (IV) fluids and inserting a nasogastric (NG) tube Administering oral bicarbonate and testing the patient's gastric pH level Performing a fecal occult blood test and administering IV calcium gluconate Starting parenteral nutrition and placing the patient in a high-Fowler's position

Providing intravenous (IV) fluids and inserting a nasogastric (NG) tube A perforated peptic ulcer requires IV replacement of fluid losses and continued gastric aspiration by NG tube. Oral bicarbonate would not be given, because the client would be nothing by mouth, and gastric pH testing is not a priority. Calcium gluconate is not a medication directly relevant to the patient's suspected diagnosis and parenteral nutrition is not a priority in the short term.

The nurse is providing care to a patient with cholecystitis that is experiencing severe nausea and vomiting. The nurse should include what intervention? Encouraging exercise Assisting with ambulation Assisting with repositioning Providing oral care every two hours

Providing oral care every two hours Patients with cholecystitis may have severe nausea and vomiting. Therefore, the nurse should give frequent oral care to the patient to avoid discomfort. Encouraging exercise, assisting with ambulation, and repositioning are not interventions that address the problem of severe nausea and vomiting. p. 1040

Hep C transmission

RNA virus transmitted percutaneously IV drug use High-risk sexual behaviors Occupational exposure Dialysis Perinatal exposure Blood transfusions before 1992

Hep A transmission

RNA virus transmitted via fecal-oral route Contaminated food or drinking water

A patient who was using antacids for a week reports having an upset stomach and liquid bowel movements. After reviewing the patient's record, the nurse asks the patient to immediately stop taking the antacid. Which over-the-counter antacid is the patient likely consuming? Calcium carbonate [Tums] Aluminum hydroxide [Amphojel] Sodium bicarbonate [Alka-Seltzer] Magnesium hydroxide [Milk of Magnesia]

Magnesium hydroxide [Milk of Magnesia] Magnesium hydroxide [Milk of Magnesia] is an antacid that contains magnesium and is known to cause diarrhea if used over the long term, so the nurse should ask the patient to immediately stop taking the antacid. Aluminum hydroxide [Amphojel] is an aluminum-based antacid that can cause constipation and other severe conditions such as hypercalcemia. The combination of magnesium- and aluminum-based antacids can nullify the adverse effects caused by each. Calcium carbonate [Tums] is a calcium-based antacid known to cause nausea and vomiting, as well as severe conditions such as kidney stones. Sodium bicarbonate [Alka-Seltzer] is a sodium-based antacid known to cause metabolic alkalosis if used over the long term. It also causes adverse effects in patients with cardiac problems.

A patient has been taking aluminum hydroxide [Amphojel] to treat gastric hyperacidity for a few days. The patient reports being constipated. Which drug will the provider likely order in addition to aluminum hydroxide [Amphojel]? Sodium citrate [Citra pH] Calcium carbonate [Tums] Aluminum carbonate [Basaljel] Magnesium hydroxide [Milk of Magnesia]

Magnesium hydroxide [Milk of Magnesia] The constipation caused by the aluminum-based antacid is neutralized when taken in combination with a magnesium-based antacid such as magnesium hydroxide [Milk of Magnesia], which acts as a laxative. Administration of aluminum carbonate [Basaljel] and aluminum hydroxide [Amphojel] would aggravate constipation. Sodium-based antacids such as sodium citrate [Citra pH] along with aluminum hydroxide [Amphojel] have no effect in relieving the effects of the latter. Calcium-based antacids such as calcium carbonate [Tums] also cause constipation, and hence their use will not alleviate the patient's discomfort.

Hep C acute phase incubation period symptoms & duration

Malaise Anorexia Weight loss Fatigue Nausea/vomiting Abdominal discomfort Distaste for cigarettes ↓ Sense of smell Headache Low-grade fever Arthralgias Skin rashes Maximal infectivity; lasts 1-4 months

Which medication for peptic ulcer disease causes hallucinations? Nizatidine Sucralfate Omeprazole Metoclopramide

Metoclopramide Metoclopramide is a prokinetic agent that causes central nervous system side effects such as hallucinations and anxiety. Nizatidine is a histamine (H 2)-receptor blocker that causes abdominal pain, headache, diarrhea, and constipation. Sucralfate is an antiulcer medication that causes constipation. Omeprazole is a proton pump inhibitor that causes nausea, abdominal pain, headache, diarrhea, and flatulence. p. 934

Instruction by the nurse regarding alcohol abstinence is essential when a patient will be discharged taking which medication? Tetracycline Metronidazole Clarithromycin Bismuth subsalicylate

Metronidazole A disulfiram-like [Antabuse] reaction can occur if metronidazole is used with alcohol; therefore, alcohol must be avoided during treatment with this drug. Although the use of alcohol is not promoted in patients who take tetracycline, clarithromycin, or bismuth subsalicylate, it does not create an adverse reaction.

Which medication is contraindicated during pregnancy? Cimetidine [Tagamet] Misoprostol [Cytotec] Lansoprazole [Prevacid] Calcium carbonate [Tums]

Misoprostol [Cytotec] The use of misoprostol [Cytotec] is contraindicated in pregnant women because it is classified under pregnancy category X and is known for its abortifacient property. Cimetidine [Tagamet] is an H 2 receptor antagonist, and lansoprazole [Prevacid] is a proton pump inhibitor. Both are classified under pregnancy category B and can be safely administered to pregnant women. Calcium carbonate [Tums] is a calcium-containing antacid classified under pregnancy category A. Drugs that are classified under pregnancy categories A and B are safe for pregnant woman and cause no adverse effects.

The nurse is developing a plan of care for a patient with acute pancreatitis. What nursing actions should be included in the plan of care? Observe for skin irritation. Monitor electrolyte levels. Assess respiratory function. Provide psychologic support. Observe for muscular twitching.

Monitor electrolyte levels. Assess respiratory function. Observe for muscular twitching. Frequent gastric suctioning is performed in patients with severe acute pancreatitis, which may result in decreased levels of the electrolytes, sodium, potassium, calcium, and chloride; therefore, electrolyte levels should be monitored. The nurse should assess respiratory function by determining the lung sounds and oxygen saturation levels. Hypocalcemia results in muscular twitching and jerking movement, so the nurse should look for muscle twitching in the patient. Skin irritation is seen in patients who undergo radiation therapy, so the nurse need not observe for skin irritation. Patients with chronic conditions and incurable diseases require psychologic support in order to cope.

A patient has a prescription to receive famotidine (Pepcid) 30 mg intravenous piggyback (IVPB) or intravenous push (IVP) q12h. Available is a vial containing 10 mg/mL. How many milliliters should the nurse draw up to administer this dose? 0.2 mL 0.75 mL 3 mL 5 mL

Multiply 10 by x and multiply 30 × 1 to yield 10x = 30. Divide 30 by 10 to yield 3 mL.

The patient history indicates the patient was taking ondansetron at home before admission. The nurse inquires as to the effectiveness of this medication in treating which symptom? Headache Nausea Pain Leg cramps

Nausea Ondansetron is an antiemetic. The nurse would inquire as to its effectiveness in reducing the patient's nausea. Ondansetron will not treat headaches, pain, or leg cramps.

populations at risk of CRC

Need earlier and more frequent screening-Those with first-degree relative in whom CRC was diagnosed before age 60 & Those with two first-degree relatives with CRC

A patient who has gastroesophageal reflux disease (GERD) is taking magnesium hydroxide (milk of magnesia). Which outcome should a nurse expect if the medication is achieving the desired therapeutic effect? Reduced duodenal pH Neutralized gastric acid Reduced stomach motility Increased barrier to pepsin

Neutralized gastric acid Antacids work by neutralizing, absorbing, or buffering gastric acid, which raises the gastric pH above 5. For patients with GERD, antacids can produce symptomatic relief. An increased barrier to pepsin is an effect of sucralfate [Carafate]. Reduced stomach motility is not an effect of milk of magnesia. A reduced duodenal pH would result in a more acidic environment. pp. 958-959

An 80-year-old patient with a history of renal insufficiency recently was started on cimetidine. Which assessment finding indicates that the patient may be experiencing an adverse effect of the medication? +3 pitting edema Pain with urination New onset of disorientation to time and place Heart rate changes from a baseline of 70 to 80 beats/min to 110 to 120 beats/min

New onset of disorientation to time and place Effects on the central nervous system are most likely to occur in elderly patients who have renal or hepatic impairment. Patients may experience confusion, hallucinations, lethargy, restlessness, and seizures. Pitting edema, dysuria, and tachycardia are not adverse effects of cimetidine. p. 953

drug therapy GI bleed acid reducers

No proven ability to control active bleeding Acidic environment can alter platelet function and clot stabilization Histamine-2 receptor (H2R) blockers (given IV)- Give 1 hour apart from antacids, Inhibit action of histamine at H2 receptors and decrease HCl acid secretion (Cimetidine (Tagamet) Ranitidine (Zantac)) Proton pump inhibitors (PPIs)-More effective than H2RB, Suppresses gastric secretion by inhibiting H+, K+, ATPase enzyme system, inhibits gastric acid pump (Pantoprazole (Protonix), Esomeprazole (Nexium))

The nurse is assisting a patient who has been admitted with severe abdominal pain. Suddenly, the patient vomits a large amount of emesis that looks similar to coffee grounds. Which action by the nurse is a priority? Ask the patient about the timing of the last meal. Offer the patient sips of water to prevent dehydration. Monitor the patient for any further episodes of nausea and vomiting. Notify the primary health care provider about the patient's condition.

Notify the primary health care provider about the patient's condition. Vomitus with a "coffee ground" appearance is related to gastric bleeding, in which blood changes to dark brown as a result of its interaction with hydrochloric acid. The primary health care provider needs to be notified immediately about this change in the patient's condition. Asking the patient about the timing of the last meal and monitoring the patient are appropriate, but not the priority. The nurse should not offer water just in case the patient may have to have a diagnostic study that requires nothing by mouth (NPO) status.

Hematemesis

Obvious GI bleed: Bloody vomitus , Appears fresh, bright red blood or "coffee grounds"

A nurse administers which medication to inhibit an enzyme that makes gastric acid in a patient who has a duodenal ulcer? Ranitidine [Zantac] Famotidine [Pepcid] Misoprostol [Cytotec] Omeprazole [Prilosec]

Omeprazole [Prilosec] Omeprazole causes irreversible inhibition of the proton pump, the enzyme that generates gastric acid. It is a powerful suppressant of acid secretion. Famotidine and ranitidine block histamine 2 receptors on parietal cells. Misoprostol protects against ulcers caused by nonsteroidal anti-inflammatory drugs (NSAIDs) by stimulating the secretion of mucus and bicarbonate to maintain submucosal blood flow. p. 954

A patient with peptic ulcers reports pain in the stomach. A gastric tissue biopsy reveals the presence of Helicobacter pylori infection. What should be the first-line treatment for the patient? Cimetidine [Tagamet] combined with sucralfate [Carafate] and clarithromycin [Biaxin] Omeprazole [Prilosec] combined with clarithromycin [Biaxin]and sucralfate [Carafate] Cimetidine [Tagamet] combined with metronidazole [Flagyl] and ketoconazole [Nizoral] Omeprazole [Prilosec] combined with metronidazole [Flagyl] and clarithromycin [Biaxin]

Omeprazole [Prilosec] combined with metronidazole [Flagyl] and clarithromycin [Biaxin] The first-line treatment for peptic ulcers associated with Helicobacter pylori infections includes a 10- to 14-day course of a proton pump inhibitor (PPI) and the antibiotics clarithromycin [Biaxin] and either amoxicillin [Amoxil] or metronidazole [Flagyl]. Omeprazole [Prilosec] is a PPI that decreases the levels of hydrochloric acid in the stomach, and the antibiotics metronidazole [Flagyl] and clarithromycin [Biaxin] inhibit the growth of H. pylori. Cimetidine [Tagamet] is an H 2 receptor blocker, which is less safe than PPIs and is not preferred. Sucralfate [Carafate] decreases the absorption of PPIs and so should not be taken with them.

CRC clinical manifestations

Onset insidious Advanced stages: Change in bowel habits, Unexplained weight loss, Vague abdominal pain, Weakness and fatigue, Iron-deficiency anemia and occult bleeding Left side cancer symptoms appear earlier Cancerous lesions: Rectal bleeding - most common, Alternating constipation and diarrhea, Change in stool caliber, Narrow, ribbon-like, Sensation of incomplete evacuation, Obstruction

prazole

PPI (omeprazole)

A patient reports severe abdominal pain, nausea, and vomiting. The nurse assesses that the patient has a mass in the upper region of the abdomen. The laboratory reports show the white blood cell (WBC) count as 12,000 cells/µL. What does the nurse anticipate that the patient will be treated for? Pseudocyst Cullen's sign Spider angioma Pancreatic abscess

Pancreatic abscess A normal count of WBC is 4,500 to 10,000 cells/µL. The laboratory reports reveal an increased WBC count, which indicates that the patient has leukocytosis. Abdominal mass, abdominal pain, nausea, vomiting, and leukocytosis are manifestations of pancreatic abscess. Therefore the primary health care provider suspects that the patient has pancreatic abscess. Though pseudocysts have the same manifestations as those of pancreatic abscess, the laboratory findings of pseudocysts include elevated serum amylase rather than increased WBC. Cullen's sign is manifested by discoloration of the abdominal wall and the periumbilical region. Spider angioma is a manifestation of liver cirrhosis, wherein the patient has a dilated blood vessel with a red center that branches like a spider web.

A patient reports nausea, a 5 kg weight loss in six days, and pain in the abdomen radiating to the back. After reviewing the magnetic resonance imaging (MRI) report, a primary health care provider orders a CA 19-9 test to confirm the diagnosis. Which condition is suspected? Cholecystitis Cholelithiasis Pancreatic cancer Acute pancreatitis

Pancreatic cancer Nausea, rapid weight loss, and dull abdominal pain that radiates to the back are clinical manifestations of pancreatic cancer. The CA 19-9 is a cancer-associated antigen that is elevated in patients with pancreatic cancer. Therefore the primary health care provider suspects pancreatic cancer. Nausea, vomiting, fever, and jaundice are characteristics of cholecystitis. Abdominal pain accompanied by tachycardia and perspiration are characteristics of cholelithiasis. Moreover, diagnostic studies such as abdomen ultrasound, liver function tests, and serum amylase are ordered to confirm disorders of the biliary tract such as cholecystitis and cholelithiasis. In acute pancreatitis, the cancer-associated antigen CA 19-9 may not be elevated.

When assessing a patient with a gastrointestinal disorder, the nurse finds an absence of bowel sounds and peristalsis. The assessment findings are indicative of what condition? Volvulus Borborygmi Paralytic ileus Pseudopolyps

Paralytic ileus Lack of intestinal peristalsis and an absence of bowel sounds indicate paralytic ileus, which occurs due to paralysis of intestinal muscles. Volvulus is an intestinal obstruction that occurs by the bowel twisting around a focal point. Borborygmi are audible abdominal sounds due to hyperactive intestinal motility. Pseudopolyps are tongue-like projections into the bowel lumen. p. 982

Paracentesis nursing actions

Patient voids immediately before High Fowler's position or sitting on side of bed Monitor for fluid and electrolyte imbalances Monitor dressing for bleeding/leakage

The use of a sodium-based antacid is contraindicated in which group of patients? Patients with infections Patients with renal failure Patients with heart failure Patients with low bone density

Patients with heart failure Sodium-based antacids have adverse effects on patients with heart failure because the sodium content in these formulations is very high. Sodium-based antacids can be taken by patients suffering from renal disorders because they are easily excreted. Patients with infection can also consume sodium-based antacids in low doses since the acids protect the system from infection. Patients with a low bone density are advised not to consume aluminum-based antacids. p. 958

drug therapy: Hep C, severe Hep B (none for Hep A)

Pegylated interferon w/ ribavarin (teratogenic) to reduce progression to chronic infection (Rebetol, Copegus) chronic hep B: Interferon Multiple effects on viral replication Two forms: standard and pegylated Must be administered subcutaneously Varied response to treatment Side effects: Flu-like symptoms, depression, irritability, insomnia, neutropenia, thrombocytopenia Nucleoside/nucleotide analogs Decrease viral replication by inhibiting normal DNA synthesis (Lamivudine (Epivir), Adefovir (Hepsera), Entecavir (Baraclude), Telbivudine (Tyzeka), Tenofovir (Viread))

Which medication does the nurse expect to be beneficial to the patient who works outdoors, has high levels of emotional stress, and complains of lip lesions and painful ulcers? Ampicillin Penciclovir Prednisone Amphotericin B

Penciclovir Lip lesions and painful ulcers caused by herpes simplex infection and triggered by excessive exposure to sunlight and emotional stress can be treated by antiviral drugs such as penciclovir. Ampicillin is an antibiotic used in the treatment of parotitis and Vincent's infection. Prednisone is a corticosteroid used in the treatment of aphthous stomatitis. Amphotericin B is an antifungal agent used in the treatment of oral candidiasis.

A patient who is receiving treatment for cholelithiasis experiences severe nausea and six episodes of vomiting. The nurse identifies that which intervention will be beneficial to the patient? Administrating lactulose Administrating rifaximin Performing gastric decompression Performing endotracheal intubation

Performing gastric decompression Gastric decompression is performed to remove the gastric contents via a nasogastric tube to prevent aspiration of food contents and reduce the incidence of nausea and vomiting. Lactulose is a laxative and is used to treat constipation. Rifaximin is an antibiotic prescribed for patients with hepatic encephalopathy. Endotracheal intubation is a means of providing ventilation to the patient.

The nurse should associate which factor(s) with the pathophysiology of peptic ulcer disease? Poor submucosal gastric blood flow Increased synthesis of prostaglandins Presence of Zollinger-Ellison syndrome Reduced stomach production of bicarbonate Gastrointestinal tract colonized with Haemophilus influenzae

Poor submucosal gastric blood flow Presence of Zollinger-Ellison syndrome Reduced stomach production of bicarbonate Peptic ulcers develop when an imbalance exists between mucosal defensive factors and aggressive factors. Sufficient blood flow to cells of the gastrointestinal mucosa is needed to maintain integrity. In Zollinger-Ellison syndrome, hypersecretion of acid alone causes ulcers by overcoming mucosal defenses. Bicarbonate is needed to neutralize hydrogen ions. Prostaglandins are needed to stimulate mucus and bicarbonate to maintain mucosal blood flow. Helicobacter pylori, not Haemophilus influenzae, is a bacillus that can disrupt the protective mucosal layer when colonized in the GI tract. pp. 947-948

Complications of decompensated cirrhosis

Portal hypertension:Increased portal venous pressure, Splenomegaly, Large collateral veins, Ascites, Gastric and esophageal varices Esophageal varices: Complex of tortuous veins at lower end of esophagus, Very fragile Bleed easily; bleeding can be life-threatening and gastric varices: Upper portion of stomach Peripheral edema: ↓ Colloidal oncotic pressure from impaired liver synthesis of albumin, ↑ Portacaval pressure from portal hypertension, Occurs as ankle/presacral edema and ascites: Accumulation of serous fluid in peritoneal or abdominal cavity, Abdominal distention with weight gain, Common manifestation of cirrhosis Hepatic encephalopathy: Neurotoxic effects of ammonia, Abnormal neurotransmission, Astrocyte swelling, Inflammatory cytokines Liver unable to convert increased ammonia, Ammonia crosses blood-brain barrier, Changes in neurologic and mental responsiveness, Impaired consciousness and/or inappropriate behavior, Sleep disturbances to lethargy to coma, Asterixis , Fetor hepaticus Hepatorenal syndrome: Renal failure with azotemia, oliguria, and intractable ascites, No structural abnormality of kidneys, Portal hypertension → vasodilation → renal vasoconstriction, Treat with liver transplantation

Varices treatment

Prevent bleeding/hemorrhage Avoid alcohol, aspirin, and irritating foods Screen for presence with endoscopy Nonselective β-blocker If bleeding occurs, stabilize patient, manage airway, provide IV therapy and blood Drug therapy Octreotide (Sandostatin) Vasopressin (VP, terlipressin) with nitroglycerin (NTG) Endoscopic therapy Band ligation Sclerotherapy

PUD potential abnormal assessment findings

Projectile vomiting Client reports burning 2 hours after eating Episode of coffee-ground emesis Boardlike abdomen with shoulder pain

A patient reports having a dry mouth and asks for something to drink. The nurse recognizes that this symptom most likely can be attributed to a common adverse effect of which medication that the patient is taking? Digoxin Cefotetan Famotidine Promethazine

Promethazine A common adverse effect of promethazine, an antihistamine/antiemetic agent, is dry mouth; another is blurred vision.

Deep muscle injection is the preferred route of administration of this drug used to treat nausea and vomiting because of its necrotizing effect on tissue when infiltration occurs. Methergine Prochlorperazine Metoclopramide Promethazine

Promethazine When promethazine is administered intravenously (IV), it can leak out from the vein and cause serious damage to surrounding tissue. Deep muscle injection is the preferred route of injection administration. Methergine is used to control postpartum hemorrhage. Prochlorperazine and metoclopramide are not reported to have as damaging an effect on tissue when administered IV.

The nurse is caring for a patient who is undergoing radiation therapy for pancreatic cancer. What nursing action is most beneficial for this patient? Giving a low-fiber diet Ignoring persistent skin irritation Increasing dairy products in the diet Providing brightly-colored and attractive food

Providing brightly-colored and attractive food Patients undergoing radiation therapy experience loss of appetite. Serving brightly colored and attractive food may increase the patient's appetite. Giving low-fiber foods can result in constipation in the patient. Radiation therapy causes skin irritation, which should be brought to the primary health care provider's notice. Increasing dairy products in the diet may not improve the patient's appetite.

Hepatic encephalopathy treatment

Reduce ammonia formation Lactulose (Cephulac), which traps ammonia in gut Rifaximin (Xifaxan) antibiotic Prevent constipation Treatment of precipitating cause Control GI bleeding Remove blood from GI tract

The nurse is caring for a patient diagnosed with suspected posterior duodenal ulcer penetration of the pancreas. Which diagnostic test does the nurse anticipate to be most beneficial to the patient? Rapid urease testing Liver enzyme studies Serum amylase determination Complete blood count test (CBC)

Serum amylase determination A serum amylase determination test is performed to determine the pancreatic function when posterior duodenal ulcer penetration of the pancreas is suspected. Rapid urease testing is done to identify the presence of Helicobacter pylori infection. Liver enzyme studies are done to detect liver problems that may complicate ulcer treatment. A CBC test is a blood count test used to detect anemia secondary to ulcer bleeding.

Diagnostics chronic pancreatitis

Serum amylase/lipase levels: May be ↑ slightly or not at all ↑ Serum bilirubin level ↑ Alkaline phosphatase level Mild leukocytosis ↑ Sedimentation rate ERCP CT, MRI, MRCP, abdominal and/or endoscopic ultrasonography Stool samples for fat content ↓ Fat-soluble vitamin and cobalamin levels Glucose intolerance/diabetes Secretin stimulation test

A patient is prescribed ranitidine [Zantac] for the treatment of peptic ulcers. To ensure drug safety, what should the nurse assess before administering the drug? Blood glucose levels Blood pressure levels Serum antibody levels Serum creatinine levels

Serum creatinine levels Assessment of serum creatinine levels is useful for determining the kidney function that is required to prescribe H 2 receptor antagonist drugs such as ranitidine [Zantac]. Monitoring blood glucose and blood pressure gives a general idea about the patient's well-being. Serum antibody levels detect possible infections in the patient.

A 54-year-old patient admitted with cancer has not been able to eat because of nausea. Which strategies should the nurse use to increase the patient's intake? Serve foods that are warm to hot in temperature. Administer antiemetics one hour before meals to prevent nausea. Offer the patient foods, such as cooked cereal and soft or canned fruits. Offer the patient meats and foods with mild spices. Offer a diet that appeals to the patient's preferences.

Serve foods that are warm to hot in temperature. Administer antiemetics one hour before meals to prevent nausea. Offer a diet that appeals to the patient's preferences. pp. 927-928

A patient with a rectal tumor has undergone an abdominal-perineal resection (APR). The nurse should monitor the patient for what postoperative complications? Hepatotoxicity Sexual dysfunction Delayed wound healing Persistent perineal sinus tracts Upper respiratory tract infection

Sexual dysfunction Delayed wound healing Persistent perineal sinus tracts Abdominal-perineal resection (APR) is a colorectal cancer surgery in which both the tumor and the entire rectum are removed, and the patient has a permanent colostomy. Abdominal-perineal resection (APR) involves complications such as sexual dysfunction, delayed wound healing, and persistent perineal sinus tracts. Hepatotoxicity and upper respiratory tract infections are complications associated with immunomodulators. p. 988

A female patient expresses her concern about becoming pregnant while her partner is on ribavirin therapy for chronic hepatitis C. What should the nurse advise the patient? She can plan pregnancy now. She should avoid getting pregnant now. She should not get pregnant with this partner ever. She should avoid any sexual intercourse after conception.

She should avoid getting pregnant now. Any woman who is on ribavirin or whose male partner is on is on ribavirin should avoid pregnancy during treatment. The pregnancy can be planned after the treatment is complete. She can get pregnant with this partner, but not while on treatment. Avoiding intercourse after conception is not necessary.

A patient is on nonsteroidal antiinflammatory drugs (NSAIDs). Which complication does the nurse anticipate? Achalasia Duodenal ulcer Stomach cancer Silent peptic ulcer

Silent peptic ulcer Silent peptic ulcers show no symptoms of ulcer disease until the presentation of their final, fatal illness. The ulcers occur in older patients or patients who take NSAIDs. Achalasia is a primary motility disorder characterized by the absence of peristalsis and impaired relaxation of the lower esophageal sphincter (LES) in response to swallowing. Duodenal ulcers occur due to excessive alcohol ingestion and smoking that result in increased hydrochloric acid secretion. Stomach cancer begins with a nonspecific mucosal injury as a result of infection, autoimmune-related inflammation, and tobacco use.

What are the late symptoms of cancer of the tongue? Toothache Slurred speech Increased salivation Cancerous lesions on the tongue Limited movement of the tongue

Slurred speech Cancerous lesions on the tongue p. 929

Occult bleeding

Small amounts of blood in gastric secretions, vomitus, or stools, Undetectable by appearance, Detectable by guaiac test

The nurse is caring for a patient with liver cirrhosis. Which medication should be used with caution in this patient? Sodium citrate Magnesium oxide Calcium carbonate Aluminum hydroxide

Sodium Citrate Antacids high in sodium citrate need to be used with caution in a patient with liver cirrhosis because they interact with and reduce the absorption of the medications prescribed for liver cirrhosis. Magnesium oxide should be used with caution on patients with renal failure. Calcium carbonate should be cautiously prescribed to patients with hypercalcemia. Aluminum hydroxide should be used with caution on patients with hemorrhoids. p. 946

Ascites treatment

Sodium restriction Albumin Diuretics Tolvaptan (Samsca) Paracentesis Transjugular intrahepatic portosystemic shunt (TIPS)

A patient who is hospitalized for a fractured hip develops abdominal pain and is diagnosed with a gastric ulcer. Which acid-controlling drug will the primary healthcare provider likely order? Sucralfate [Carafate] Misoprostol [Cytotec] Omeprazole [Prilosec] Lansoprazole [Prevacid]

Sucralfate [Carafate] Sucralfate [Carafate] is known to act locally by forming a protective barrier (mucus) against pepsin on the surface of the ulcer, healing the ulcer. Therefore, the nurse should administer sucralfate [Carafate]. Since the patient has a fractured hip, misoprostol [Cytotec] would not be prescribed because of probable adverse effects such as diarrhea. Proton pump inhibitors such as omeprazole [Prilosec] and lansoprazole [Prevacid] are known to stimulate bone mineral loss and thus may interfere with the healing of the bone. pp. 956-957

differentiate acute & chronic ulcers

The acute ulcer (Fig. 42-9) is associated with superficial erosion and minimal inflammation. It is of short duration and resolves quickly when the cause is identified and removed. A chronic ulcer (Fig. 42-10) is one of long duration, eroding through the muscular wall with the formation of fibrous tissue. It is present continuously for many months or intermittently throughout the person's lifetime. Chronic ulcers are more common than acute erosions.

The nurse is teaching care guidelines to the caregiver of a patient with upper gastrointestinal (GI) bleeding. In the follow-up visit, the patient complains of traces of blood in the vomit. Which action of the patient's caregiver is responsible for the patient's condition? The caregiver gave aspirin to the patient on an empty stomach. The caregiver allowed the patient to drink 10 mL of alcohol once a month. The caregiver gave the patient an analgesic along with a proton pump inhibitor. The caregiver encouraged the patient to practice deep breathing exercises.

The caregiver gave aspirin to the patient on an empty stomach. Traces of blood in the vomit indicate gastrointestinal bleeding. When given on an empty stomach, aspirin irritates the gastrointestinal mucosa and causes gastrointestinal (GI) bleeding. Therefore, it should be taken along with meals or snacks. Alcohol consumption aggravates the symptoms of hyperacidity. However, drinking 10 mL of alcohol once a month will not cause GI bleeding. Co-administration of nonsteroidal antiinflammatory drugs (NSAIDs) and proton-pump inhibitors reduces the risk of bleeding. Deep breathing provides relaxation and does not cause GI bleeding. p. 956

The patient with a history of lung cancer and hepatitis C has developed liver failure and is considering liver transplantation. After the comprehensive evaluation, the nurse knows that which factor discovered may be a contraindication for liver transplantation? The patient has completed a college education. The chest x-ray showed another lung cancer lesion. The patient has been able to stop smoking cigarettes. The patient has well controlled type 1 diabetes mellitus.

The chest x-ray showed another lung cancer lesion. Contraindications for liver transplant include severe extrahepatic disease, advanced hepatocellular carcinoma or other cancer, ongoing drug or alcohol abuse, and the inability to comprehend or comply with the rigorous post-transplant course. It does not matter if the patient has a college education. The fact that the patient has quit smoking is not a contraindication for liver transplant. The patient is a well-controlled diabetic, which is not a contraindication.

Why are proton pump inhibitors (PPIs) used in the treatment of patients with bleeding due to erosive esophagitis? They affect the absorption of food. They do not alter the levels of acid. They act directly on the proton pump. They inhibit the proton pumps in all cells of the digestive system.

They act directly on the proton pump. Erosive esophagitis is a condition in which irritation is caused by acid in the esophagus. PPIs directly inhibit the hydrogen-potassium-ATPase pump, thus reducing the release of hydrogen ions that form acid. Food digestion and absorption are unaltered by these drugs. About 90% of acid secretion is stopped within 24 hours of administration, bringing relief to the patient. PPIs inhibit only the proton pump of parietal cells and reduce only gastric acids. pp. 954-955

Colorectal cancer etiology

Third most common form of cancer 9% of cancer deaths are due to CRC More common in men Highest mortality rates among African American men and women Risk increases with age Of new cases, 90% in people older than 50 Risk Factors: Diet that is...High in red or processed meat, Low in fruits and vegetables Lifestyle factors- Obesity, Physical inactivity, Alcohol, Long-term smoking Health factors-Personal history of IBD, Individual or family history of colorectal cancer, Colorectal polyps, Age Adenocarcinoma-most common type of CRC About 85% arise from adenomatous polyps

describe the procedure Billroth I: gastroduodenostomy

Used in GOO when conservative therapy is not effective/ stomach cancer suspected. Partial gastrectomy with removal of distal 2/3 of stomach and anastomosis of gastric stump to duodenum

describe the procedure Billroth II: gastrojejunostomy

Used in GOO when conservative therapy is not effective/ stomach cancer suspected. Partial gastrectomy with removal of distal 2/3 of stomach and anastomosis of gastric stump to jejunum

Angiography

Used to diagnose only when endoscopy cannot be done Invasive procedure May not be appropriate for high-risk or unstable patient Catheter placed into left gastric or superior mesenteric artery until site of bleeding is discovered

The primary health care provider is severing the vagus nerve in a patient with a peptic ulcer that is unresponsive to medical management. Which surgical procedure should the nurse expect for this patient? Billroth I Billroth II Vagotomy Pyloroplasty

Vagotomy A vagotomy is severing of the vagus nerve either totally (truncal) or selectively (highly selective vagotomy). These procedures are done to decrease gastric acid secretion. Billroth I involves a partial gastrectomy with the removal of two-thirds of the stomach and the anastomosis of the gastric stump to the duodenum. Billroth II is partial gastrectomy with the removal of two-thirds of the stomach and the anastomosis of the gastric stump to the jejunum. Pyloroplasty consists of surgical enlargement of the pyloric sphincter to facilitate easy passage of contents from the stomach.

Diverticular Colon Disease in Adults

Very common in developed countries and prevalence increases with age Thickening of muscle wall and shortening of taenia with a resulting accordion-like bunching of the folds. Western diet low in fiber

Hepatitis causes

Viral (most common) Drugs (alcohol) Chemicals Autoimmune diseases Metabolic abnormalities

After administering a dose of metoclopramide to the patient, the nurse determines that the medication has not been effective when which symptom is noted? Migraine headache Vertigo Vomiting Diarrhea

Vomiting Metoclopramide is classified as a cholinergic and antiemetic medication. If it is effective, the patient's nausea should resolve. Metoclopramide is not effective in the treatment of migraine, vertigo, or diarrhea.

Post op complications: Billroth I & II

Will need cobalamin (B12) injections for life to prevent anemia Dumping syndrome-20% of clients experience after surgery, result of surgical removal of large portion of stomach and pyloric sphincter, ↓ ability of stomach to control amount of chyme entering small intestine: Large bolus of hypertonic fluid enters intestine & ↑ fluid drawn into bowel lumen, Occurs at end of meal or 15 to 30 minutes after eating, Symptoms include: Weakness, sweating, palpitations, dizziness, cramps, borborygmi, urge to defecate, and last no longer than an hour

What causes bleeding in pancreatitis

activation of trypsinogen to trypsin

antacids mechanism of action, side effects

aluminum carbonate (Basaljel) aluminum hydroxide (Amphojel) calcium carbonate (Tums, Titralac) magnesium oxide (MagOx) sodium bicarbonate (Alka-Seltzer) sodium citrate (Bicitra) Gelusil, Maalox, Mylanta, Aludrox aluminum/magnesium trisilicate (Gaviscon) Neutralizes HCl acid Taken 1-3 hr after meals and at bedtime Aluminum hydroxide: Constipation, phosphorus depletion with chronic use Calcium carbonate: Constipation or diarrhea, hypercalcemia, milk-alkali syndrome, renal calculi Magnesium preparations: Diarrhea, hypermagnesemia Sodium preparations: Milk-alkali syndrome if used with large amounts of calcium; use with caution in patients on sodium restrictions

azole

antifungal (metronidazole)

cholinergic mechanism of action, side effects

bethanechol (Urecholine) ↑ LES pressure, improves esophageal emptying, increases gastric emptying Lightheadedness, syncope, flushing diarrhea, stomach cramps, dizziness

Bleeding from the esophagus is most likely due to

chronic esophagitis (GERD, Mucosa-irritating drugs Aspirin, NSAIDs, corticosteroids, Alcohol, Cigarettes), Mallory-Weiss tear, or esophageal varices.

Most common causes of cirrhosis

chronic hepatitis C and alcohol-induced liver disease:Protein malnutrition, Environmental factors, Chronic inflammation and cell necrosis, Synergistic factors accelerate damage

H2 receptor blockers mechanism of action, side effects

cimetidine (Tagamet) famotidine (Pepcid) nizatidine (Axid) ranitidine (Zantac) Blocks the action of histamine on the H2-receptors to ↓ HCl acid secretion ↓ Conversion of pepsinogen to pepsin ↓ Irritation of the esophageal and gastric mucosa Headache, abdominal pain, constipation, diarrhea

Pancreatic abscess

complication r/t acute pancreatitis Collection of pus Results from extensive necrosis May perforate Upper abdominal pain, mass, high fever, leukocytosis Surgical drainage

psuedocyst

complication r/t acute pancreatitis Fluid, enzyme, debris, and exudates surrounded by wall Abdominal pain, palpable mass, nausea/vomiting, anorexia Detected with imaging Resolves spontaneously or may perforate and cause peritonitis Surgical or endoscopic drainage

A patient is prescribed an aluminum-containing antacid for hyperacidity. The nurse should inform the patient about which possible adverse effect? Diarrhea Flatulence Constipation Muscle twitching

constipation Formulations containing aluminum or calcium cause constipation. Aluminum, used to reduce gastric acid, binds to phosphate and may lead to hypercalcemia. Early hypercalcemia is characterized by constipation. Diarrhea is an adverse effect of magnesium-containing antacids. Flatulence is due to the accumulation of gas in the intestine and is not related to the intake of aluminum-containing antacids. Muscle twitching is due to metabolic alkalosis resulting from the excessive use of sodium bicarbonate antacids

PPI mechanism of action side effect

dexlansoprazole (Dexilant) esomeprazole (Nexium) lansoprazole (Prevacid) omeprazole (Prilosec) pantoprazole (Protonix) rabeprazole (AcipHex) ↓ HCl acid secretion by inhibiting the proton pump (H+ -K+ -ATPase) responsible for the secretion of H+ ↓ Irritation of the esophageal and gastric mucosa Headache, abdominal pain, nausea, diarrhea, vomiting, flatulence • Long-term use or high doses of PPIs may increase the risk of fractures of hip, wrist, and spine. • Lower doses or shorter duration of therapy should be considered. • PPIs are associated with increased risk of C. difficile infection in hospitalized patients.

Stomach cancer clinical manifestations

early stages: Indigestion and stomach discomfort bloated feeling after eating, Mild nausea, Loss of appetite, Heartburn advanced: Discomfort in upper or middle part of abdomen, Blood in stool which appears as black, tarry, Vomiting or vomiting blood, Weight loss, Pain or bloating in stomach after eating, Weakness or fatigue associated with mild anemia (a deficiency in red blood cells).

cirrhosis diet

for pt's W/O complications: High in calories (3000 cal/day) ↑ Carbohydrate Moderate to low fat Protein restriction rarely justified Protein supplements for protein-calorie malnutrition Low-sodium diet for patient with ascites and edema

Somatostatin or somatostatin analog octreotide (Sandostatin)

may be used when endoscopy is not available -Used for upper GI bleeding -Reduces blood flow to GI organs and acid secretion -Given in IV boluses for 3-7 days after onset of bleeding

prokinetic agents mechanism of action, side effects

metoclopramide (Reglan) Blocks effect of dopamine ↑ Gastric motility and emptying Reduces reflux CNS side effects ranging from anxiety to hallucinations Extrapyramidal side effects (tremor and dyskinesias similar to Parkinson's disease)

prostaglandin mechanism of action, side effects

misoprostol (Cytotec) Protects lining of stomach Cytoprotective: Increases production of gastric mucus and mucosal secretion of bicarbonate Antisecretory: ↓ HCl acid secretion Abdominal pain, diarrhea, GI bleeding, uterine rupture if pregnant

For the patient at risk for gastric ulcers caused by NSAID use what drug may be prescribed?

misoprostol (Cytotec) -prostaglandin analogue, Prostaglandin stimulates secretion of mucous and bicarb, promotes submucosal blood flow, suppress gastric acid secretion, associated with adverse effects such as gastrointestinal distress, bleeding in the vagina, and headache, contraindicated in pregnant women because it is classified under pregnancy category X and is known for its abortifacient property.

Melena

obvious GI bleed: Black, tarry stools Caused by digestion of blood in GI tract Black appearance—due to iron The longer the passage of blood through intestines, the darker the stool color, caused by breakdown of Hgb and release of iron

cirrhosis clinical manifestations

onset insidious, sometimes early symptom is fatigue late symptoms: Jaundice, peripheral edema, ascites Skin, hematologic , endocrine, and neurologic disorders

The nurse is caring for a patient who presents with burning pain in the midepigastric region five hours after eating. The patient's medical history reveals pancreatic disease. Which condition does the nurse suspect? Gastric ulcer Acute gastritis Duodenal ulcer Stomach cancer

p. 944 Duodenal ulcer

S/S of colorectal cancer by location

ppt slide 8

Describe GI bleeds from arterial source

profuse, and the blood is bright red. The bright red color indicates that the blood has not been in contact with gastric HCl acid secretion

h. pylori treatment: duration 10-14 days, eradication 85%

quadruple drug therapy PPI* bismuth† tetracycline metronidazole (Flagyl)

antiulcer mechanism of action, side effects

sucralfate (Carafate) Acts to form a protective layer and serves as a barrier against acid, bile salts, and enzymes in the stomach Constipation

perforation clinical manifestations

sudden and dramatic in onset. During the initial phase (0 to 2 hours after perforation) the patient experiences sudden, severe upper abdominal pain that quickly spreads throughout the abdomen. The pain radiates to the back and is not relieved by food or antacids. The abdomen appears rigid and boardlike as the abdominal muscles attempt to protect from further injury. The patient's respirations become shallow and rapid. The heart rate is elevated (tachycardia), and the pulse is weak. Bowel sounds are usually absent. Nausea and vomiting may occur. Many patients report a history of ulcer disease or recent symptoms of indigestion.

H. pylori treatment:Duration 7-14 days, eradication 70-85%

triple drug therapy proton pump inhibitor (PPI)* amoxicillin clarithromycin (Biaxin)

Metoclopramide (Reglan) considerations

• Chronic use or high doses of metoclopramide carry the risk of tardive dyskinesia. • Tardive dyskinesia is a neurologic condition characterized by involuntary and repetitive movements of the body (e.g., extremity movements, lip smacking). • With discontinuation of the drug, the tardive dyskinesia persists. Swallowed air and reduced peristalsis after surgery can result in abdominal distention and gas pains. Early ambulation helps restore peristalsis and eliminate flatus and gas pain. Medications used to reduce gas pain include metoclopramide (Reglan) or alvimopan (Entereg) to stimulate peristalsis.

Promethazine route

• Promethazine should not be administered into an artery or under the skin because of the risk of severe tissue injury, including gangrene. • When promethazine is administered IV, it can leach out from the vein and cause serious damage to surrounding tissue. • Deep muscle injection is the preferred route of injection administration.

A patient with a chronic hepatitis C virus (HCV) infection is admitted to the hospital. What are the factors that contribute to a high risk for development of cirrhosis of the liver in this patient? Diabetes mellitus Alcohol consumption History of regular smoking Elevated levels of cholesterol Diet high in sodium and fatty foods

Diabetes mellitus Alcohol consumption Elevated levels of cholesterol Hepatitis C virus (HCV) infection is more likely than hepatitis B virus (HBV) to become chronic. An infection with HCV can lead to development of cirrhosis of the liver. People with diabetes mellitus have a compromised immune function and are at risk of developing cirrhosis. Alcohol consumption may further deteriorate the liver function and lead to development of cirrhosis of the liver. Elevated cholesterol or triglycerides suppresses liver function and may lead to progression of HCV to cirrhosis. Smoking and a high-sodium diet have no effect on hepatitis C progression because they do not affect liver function.

A nurse is planning care for a patient who has peptic ulcer disease and is taking amoxicillin [Amoxil]. The nurse is aware that amoxicillin has which mechanism of action? Coating of the ulcer crater as a barrier to acid Inhibition of an enzyme to block acid secretion Selective blockade of parietal cell histamine 2 receptors Disruption of the bacterial cell wall, causing lysis and death

Disruption of the bacterial cell wall, causing lysis and death Amoxicillin disrupts the cell wall of H. pylori, which causes lysis and death. Inhibition of an enzyme to block acid secretion is a function of the proton pump inhibitors (PPIs). Coating of the ulcer crater as a barrier to acid is an action of sucralfate [Carafate]. Selective blockade of parietal cell histamine 2 receptors is an action of the histamine 2 receptor antagonists cimetidine, ranitidine, famotidine, and nizatidine. p. 950

Following a gastrectomy performed for peptic ulcer disease, the patient has recovered and is ready for discharge. What instructions should the nurse include in discharge teaching to prevent dumping syndrome? Divide meals into six small feedings. Take fluids along with meals. Use concentrated sweets like honey, jam, and jelly. Reduce protein and fats in the diet.

Divide meals into six small feedings. To prevent dumping syndrome after gastrectomy, the patient should avoid large meals, instead dividing meals into six small meals to avoid overloading the intestines at mealtimes. Fluids should not be taken with meals. Fluids can be taken at least 30 to 45 minutes before or after meals. This helps prevent distension or a feeling of fullness. Concentrated sweets should be avoided because they sometimes cause dizziness, diarrhea, and a sense of fullness. Protein and fats should be increased in the diet to help rebuild body tissue and to meet energy needs.

A patient with a peptic ulcer is prescribed sucralfate [Carafate]. To promote drug efficiency, the nurse advises the patient to take the drug on an empty stomach. The nurse also asks the patient to avoid consuming any other form of antacid along with sucralfate [Carafate]. What is the most appropriate reason for giving these instructions? Sucralfate [Carafate] promotes the healing of ulcers. Sucralfate [Carafate] is active in acidic environments. Sucralfate [Carafate] reduces the absorption of other drugs. Sucralfate [Carafate] forms a protective barrier between the ulcer and gastric acids.

Sucralfate [Carafate] is active in acidic environments. An empty stomach has a high content of gastric acids. Sucralfate [Carafate] is an acid-controlling drug that is activated and dissociates into aluminum hydroxide and sulfate anions when it comes into contact with the acid present in the stomach. If accompanied with any other antacid, the activity of the drug decreases. It is consumed independently and rarely reduces the absorption of other drugs. It binds and accumulates the epidermal growth factor in the gastric tissue and forms a protective barrier between the ulcer and the gastric acids. Sucralfate [Carafate] promotes ulcer healing by this mechanism.

The nurse finds that the patient has a hard, painless ulcer on the upper lip. What treatment does the nurse expect the primary health care provider to prescribe? Glossectomy Surgical excision Mandibulectomy Hemiglossectomy

Surgical excision A hard, painless ulcer is a sign of oral cancer affecting the lip. Surgical excision of the ulcer is the treatment option for cancer of the lip. Glossectomy is the complete removal of the tongue. It is performed when the tongue becomes cancerous. Mandibulectomy is the removal of the mandible (lower jaw). It is done when the mandibular region is affected by oral cancer. Hemiglossectomy is the surgical removal of part of the tongue. Hemiglossectomy is prescribed if the patient has tumors on part of the tongue.

What should the nurse instruct the patient to do to best enhance the effectiveness of a daily dose of docusate sodium? Take a dose of mineral oil at the same time. Add extra salt to food on at least one meal tray. Ensure dietary intake of 10 g of fiber each day. Take each dose with a full glass of water or other liquid.

Take each dose with a full glass of water or other liquid. Docusate lowers the surface tension of stool, permitting water and fats to penetrate and soften the stool for easier passage. The patient should take the dose with a full glass of water and should increase overall fluid intake, if able, to enhance effectiveness of the medication. Mineral oil and extra salt are not recommended. Dietary fiber intake should be a minimum of 20 g daily to prevent constipation. p. 967

On a follow-up visit, the nurse finds that a patient prescribed prokinetic medication to treat nausea and vomiting has discontinued the medication. Which complication does the nurse expect to be present? Xerostomia Somnolence Tardive dyskinesia Elevated liver function tests

Tardive dyskinesia p. 925

The nurse reviews the patient's medication record and notes the following: sucralfate [Carafate] 1 gram orally four times daily before meals (7:30 AM, 11:30 AM, and 4:30 PM) and at bedtime (10:00 PM); phenytoin [Dilantin] 200 mg orally daily at 8:00 AM. Which modifications, if any, should be made to the medication regimen? The medications can be administered as ordered. The nurse should obtain a prescriber order to administer the phenytoin at 9:30 AM daily. The nurse should obtain a prescriber order for intravenous phenytoin to avoid a drug interaction. The nurse should administer the phenytoin with the 7:30 AM dose of sucralfate [Carafate], because this is more time efficient.

The nurse should obtain a prescriber order to administer the phenytoin at 9:30 AM daily. Sucralfate can impede the absorption of phenytoin; therefore, a period of 2 hours should separate these drugs. The nurse should consult the prescriber for a time administration change. Based on this information, it is not appropriate to administer the drugs as ordered, switch the phenytoin to the IV form, or administer the phenytoin with the sucralfate.

The nurse is teaching a patient about measures to prevent Salmonella poisoning. Which action indicates effective learning? The patient avoids eating cheese. The patient avoids eating half-boiled eggs. The patient avoids purchasing canned items. The patient avoids eating rewarmed meat dishes.

The patient avoids eating half-boiled eggs. Improperly cooked eggs are a source of Salmonella typhimurium. Avoiding improperly cooked eggs prevents Salmonella poisoning. Contaminated cheese contains Escherichia coli (E. coli) and therefore causes E. coli poisoning. Canned food poses a risk of botulism. Rewarmed meat contains Clostridium perfringens and may cause clostridial poisoning.

A patient is admitted to the intensive care unit after a myocardial infarction. The provider has ordered drugs to prevent gastrointestinal disorders. Which is a likely reason for administering acid-controlling drugs? The patient is prone to stress ulcers. The patient's stress levels are treated by these drugs. The general procedures for the treatment include these drugs. The adverse effects of the treatment are treated by these drugs.

The patient is prone to stress ulcers. A critically ill patient is prone to stress-related mucosal damage. Therefore, these patients should be prescribed a histamine receptor blocker or a proton pump inhibitor. The patient's stress levels can cause gastrointestinal disorders, but these drugs do not lower stress levels. The general procedures performed on the patients, such as the use of nasal tubes or feeding tubes, only increase the chances of gastrointestinal disorders. Adverse effects are specific to the type of treatment received and can be addressed by specific drugs.

An assessment reveals that a patient has renal calculi. During the assessment, the nurse learns that the patient has taken over-the-counter antacids for a long time. The patient also consumes two glasses of milk every day. What should the nurse infer from this information? The patient has high gastric pH. The patient is unable to process milk. The patient is taking a lower dose of antacid. The patient is using a calcium-based antacid.

The patient is using a calcium-based antacid. Renal stones may occur from the deposition of calcium from milk and calcium-based antacids. The fact that the patient takes an over-the-counter antacid explains the patient's digestive issues and low gastric pH (hyperacidity). A lower dose of antacid would not be effective for treating hyperacidity or cause renal stones. The fact that the patient drinks two glasses of milk every day is an indication that the patient can process milk.

compare gastric & duodenal ulcers

gastric Lesion: Superficial, smooth margins Round, oval, or cone shaped, Predominantly antrum, also in body and fundus of stomach duodenal lesion: Penetrating (associated with deformity of duodenal bulb from healing of recurrent ulcers) First 1-2 cm of duodenum gastric secretion is Normal to decreased in gastric ulcer while in a duodenal ulcer it is increased incidence: gastric is > in women peak age 50-60 yr, more common in lower socioeconomic status, ↑ With incompetent pyloric sphincter and bile reflux duodenal is > in men, but starting to increase in women, esp. postmenopausal peak age 35-45 yr, associated w/ psychologic stress, Associated with other diseases (e.g., chronic obstructive pulmonary disease, pancreatic disease, hyperparathyroidism, Zollinger-Ellison syndrome, chronic renal failure), both have increased incidence w/ smoking, drug, alcohol use clinical manifestations gastric: Burning or gaseous pressure in high left epigastrium and back and upper abdomen, Pain 1-2 hr after meals. If penetrating ulcer, aggravation of discomfort with food, Occasional nausea and vomiting, weight loss clinical manifestations duodenal: Burning, cramping, pressure-like pain across midepigastrium and upper abdomen. Back pain with posterior ulcers, Pain 2-4 hr after meals and midmorning, midafternoon, middle of night. Periodic and episodic. Pain relief with antacids and food, Occasional nausea and vomiting recurrence rate high in both complications gastric: Hemorrhage, perforation, gastric outlet obstruction, intractability complications duodenal: Hemorrhage, perforation, obstruction

when does pepsinogen activate to pepsin?

in presence of HCl acid and pH 2 to 3

why would an NG tube be used in gastritis?

in severe cases, (1) to monitor for bleeding, (2) to lavage the precipitating agent from the stomach, or (3) to keep the stomach empty and free of noxious stimuli


Kaugnay na mga set ng pag-aaral

Geography Chapter 12: South Asia

View Set

Freedom of Information Act (FOIA) exemptions

View Set

Foundations Of Western Art List #7

View Set

Chapter 13 Cost Accounting videos and exercises

View Set

ATI RN Maternal Newborn Online Practice 2019 A with NGN

View Set

Qur'an Exam 2 (Final) Study Guide

View Set

Chapter 35: Assessment of Musculoskeletal Function

View Set